K
Khách

Hãy nhập câu hỏi của bạn vào đây, nếu là tài khoản VIP, bạn sẽ được ưu tiên trả lời.

14 tháng 11 2019

\(\Leftrightarrow\Sigma_{cyc}\frac{\left(ab+bc+ca\right)^2}{2a^2+bc}\le\left(a+b+c\right)^2\)

Ta có: \(\frac{\left(ab+bc+ca\right)^2}{2a^2+bc}\le\frac{\left(ab+ca\right)^2}{2a^2}+\frac{\left(bc\right)^2}{bc}=\frac{\left(b+c\right)^2}{2}+bc\)

Tương tự rồi cộng lại ta thu được:

\(L.H.S\le\frac{\left(a+b\right)^2+\left(b+c\right)^2+\left(c+a\right)^2}{2}+ab+bc+ca\)

\(=\frac{2\left(a^2+b^2+c^2\right)+2\left(ab+bc+ca\right)}{2}+ab+bc+ca\)\(=\left(a+b+c\right)^2\)

P/s: Nhìn đơn giản chứ nó là bao nhiêu ngày suy nghĩ đấy ạ:( Chả biết đúng hay sai nữa:v

22 tháng 8 2017

Ta có:

\(\frac{1}{\left(2a+b+c\right)^2}+\frac{1}{\left(a+2b+c\right)^2}+\frac{1}{\left(a+b+2c\right)^2}\)

\(\le\frac{1}{4\left(a+b\right)\left(a+c\right)}+\frac{1}{4\left(b+a\right)\left(b+c\right)}+\frac{1}{4\left(c+a\right)\left(c+b\right)}\)

\(=\frac{2\left(a+b+c\right)}{4\left(a+b\right)\left(b+c\right)\left(c+a\right)}\)

\(=\frac{a+b+c}{2\left(a+b\right)\left(b+c\right)\left(c+a\right)}\)

Giờ ta cần chứng minh

\(\frac{a+b+c}{2\left(a+b\right)\left(b+c\right)\left(c+a\right)}\le\frac{9}{16\left(ab+bc+ca\right)}\)

\(\Leftrightarrow\left(a+b\right)\left(b+c\right)\left(c+a\right)\ge\frac{8}{9}\left(a+b+c\right)\left(ab+bc+ca\right)\)

Ta có:

\(\left(a+b\right)\left(b+c\right)\left(c+a\right)=\left(a+b+c\right)\left(ab+bc+ca\right)-3abc\)

\(\ge\left(a+b+c\right)\left(ab+bc+ca\right)-\frac{1}{9}\left(a+b+c\right)\left(ab+bc+ca\right)\)

\(=\frac{8}{9}\left(a+b+c\right)\left(ab+bc+ca\right)\)

Vậy ta có ĐPCM

18 tháng 11 2019

1. Vai trò a, b, c như nhau. Không mất tính tổng quát. Giả sử \(a\ge b\ge0\)

\(ab+bc+ca=3\). Do đó \(ab\ge1\)

Ta cần chứng minh rằng \(\frac{1}{1+a^2}+\frac{1}{1+b^2}\ge\frac{2}{1+ab}\left(1\right)\)

\(\frac{2}{1+ab}+\frac{1}{1+c^2}\ge\frac{3}{2}\left(2\right)\)

Thật vậy: \(\left(1\right)\Leftrightarrow\frac{1}{1+a^2}-\frac{1}{1+ab}+\frac{1}{1+b^2}-\frac{1}{1+ab}\ge0\\ \Leftrightarrow\left(ab-a^2\right)\left(1+b^2\right)+\left(ab-b^2\right)\left(1+a^2\right)\ge0\\ \Leftrightarrow\left(a-b\right)\left[-a\left(1+b^2\right)+b\left(1+a^2\right)\right]\ge0\\ \Leftrightarrow\left(a-b\right)^2\left(ab-1\right)\ge0\left(BĐT:đúng\right)\)

\(\left(2\right)\Leftrightarrow c^2+3-ab\ge3abc^2\\ \Leftrightarrow c^2+ca+bc\ge3abc^2\Leftrightarrow a+b+c\ge3abc\)

BĐT đúng, vì \(\left(a+b+c\right)^2>3\left(ab+bc+ca\right)=q\)

\(ab+bc+ca\ge3\sqrt[3]{\left(abc\right)^2}\)

Nên \(a+b+c\ge3\ge3abc\)

Từ (1) và (2) ta có \(\frac{1}{1+a^2}+\frac{1}{1+b^2}+\frac{1}{1+c^2}\ge\frac{3}{2}\)

Dấu ''='' xảy ra \(\Leftrightarrow a=b=c=1\)

18 tháng 11 2019

Áp dụng BĐT Cauchy dạng \(\frac{9}{x+y+z}\le\frac{1}{x}+\frac{1}{y}+\frac{1}{z}\), ta được

\(\frac{9}{a+3b+2c}=\frac{1}{a+c+b+c+2b}\le\frac{1}{9}\left(\frac{1}{a+c}+\frac{1}{b+c}+\frac{1}{2b}\right)\)

Do đó ta được

\(\frac{ab}{a+3b+2c}\le\frac{ab}{9}\left(\frac{1}{a+c}+\frac{1}{b+c}+\frac{1}{2b}\right)=\frac{1}{9}\left(\frac{ab}{a+c}+\frac{ab}{b+c}+\frac{a}{2}\right)\)

Hoàn toàn tương tự ta được

\(\frac{bc}{2a+b+3c}\le\frac{1}{9}\left(\frac{bc}{a+b}+\frac{bc}{b+c}+\frac{b}{2}\right);\frac{ac}{3a+2b+c}\le\frac{1}{9}\left(\frac{ac}{a+b}+\frac{ac}{b+c}+\frac{c}{2}\right)\)

Cộng theo vế các BĐT trên ta được

\(\frac{ab}{a+3b+2c}+\frac{bc}{b+3c+2a}+\frac{ca}{c+3a+2b}\le\frac{1}{9}\left(\frac{ac+bc}{a+b}+\frac{ab+ac}{b+c}+\frac{bc+ab}{a+c}+\frac{a+b+c}{2}\right)=\frac{a+b+c}{6}\)Vậy BĐT đc CM

ĐẲng thức xảy ra khi và chỉ khi a = b = c >0

AH
Akai Haruma
Giáo viên
2 tháng 3 2017

Bài 3)

BĐT cần chứng minh tương đương với:

\(\left ( \frac{a}{a+b} \right )^2+\left ( \frac{b}{b+c} \right )^2+\left ( \frac{c}{c+a} \right )^2\geq \frac{1}{2}\left ( 3-\frac{a}{a+b}-\frac{b}{b+c}-\frac{c}{c+a} \right )\)

Để cho gọn, đặt \((x,y,z)=\left (\frac{b}{a},\frac{c}{b},\frac{a}{c}\right)\) \(\Rightarrow xyz=1\).

BĐT được viết lại như sau:

\(A=2\left [ \frac{1}{(x+1)^2}+\frac{1}{(y+1)^2}+\frac{1}{(z+1)^2} \right ]+\frac{1}{x+1}+\frac{1}{y+1}+\frac{1}{z+1}\geq 3\) \((\star)\)

Ta nhớ đến hai bổ đề khá quen thuộc sau:

Bổ đề 1: Với \(a,b>0\) thì \(\frac{1}{(a+1)^2}+\frac{1}{(b+1)^2}\geq \frac{1}{ab+1}\)

Cách CM rất đơn giản, Cauchy - Schwarz:

\((a+1)^2\leq (a+b)(a+\frac{1}{b})\Rightarrow \frac{1}{(a+1)^2}\geq \frac{b}{(a+b)(ab+1)}\)

Tương tự với biểu thức còn lại và cộng vào thu được đpcm

Bổ đề 2: Với \(x,y>0,xy\geq 1\) thì \(\frac{1}{x^2+1}+\frac{1}{y^2+1}\geq \frac{2}{xy+1}\)

Cách CM: Quy đồng ta có đpcm.

Do tính hoán vị nên không mất tổng quát giả sử \(z=\min (x,y,z)\)

\(\Rightarrow xy\geq 1\). Áp dụng hai bổ đề trên:

\(A\geq 2\left [ \frac{1}{xy+1}+\frac{1}{(z+1)^2} \right ]+\frac{2}{\sqrt{xy}+1}+\frac{1}{z+1}=2\left [ \frac{z}{z+1}+\frac{1}{(z+1)^2} \right ]+\frac{2\sqrt{z}}{\sqrt{z}+1}+\frac{1}{z+1}\)

\(\Leftrightarrow A\geq \frac{2(z^2+z+1)}{(z+1)^2}+\frac{1}{z+1}+2-\frac{2}{\sqrt{z}+1}\geq 3\)

\(\Leftrightarrow 2\left [ \frac{z^2+z+1}{(z+1)^2}-\frac{3}{4} \right ]+\frac{1}{z+1}-\frac{1}{2}-\left ( \frac{2}{\sqrt{z}+1}-1 \right )\geq 0\)

\(\Leftrightarrow \frac{(z-1)^2}{2(z+1)^2}-\frac{z-1}{2(z+1)}+\frac{z-1}{(\sqrt{z}+1)^2}\geq 0\Leftrightarrow (z-1)\left [ \frac{1}{(\sqrt{z}+1)^2}-\frac{1}{(z+1)^2} \right ]\geq 0\)

\(\Leftrightarrow \frac{\sqrt{z}(\sqrt{z}-1)^2(\sqrt{z}+1)(z+\sqrt{z}+2)}{(\sqrt{z}+1)^2(z+1)^2}\geq 0\) ( luôn đúng với mọi \(z>0\) )

Do đó \((\star)\) được cm. Bài toán hoàn tất.

Dấu bằng xảy ra khi \(a=b=c\)

P/s: Nghỉ tuyển lâu rồi giờ mới gặp mấy bài BĐT phải động não. Khuya rồi nên xin phép làm bài 3 trước. Hai bài kia xin khiếu. Nếu làm đc chắc tối mai sẽ post.

2 tháng 3 2017

Bài 1:

Cho \(a=b=c=\dfrac{1}{\sqrt{3}}\). Khi đó \(M=\sqrt{3}-2\)

Ta sẽ chứng minh nó là giá trị nhỏ nhất

Thật vậy, đặt c là giá trị nhỏ nhất của a,b,c. Khi đó, ta cần chứng minh

\(\frac{a^2}{b}+\frac{b^2}{c}+\frac{c^2}{a}-\frac{2(a^2+b^2+c^2)}{\sqrt{ab+ac+bc}}\geq(\sqrt3-2)\sqrt{ab+ac+bc}\)

\(\Leftrightarrow\sqrt{ab+ac+bc}\left(\frac{a^2}{b}+\frac{b^2}{c}+\frac{c^2}{a}-\sqrt{3(ab+ac+bc)}\right)\geq2(a^2+b^2+c^2-ab-ac-bc)\)

\(\Leftrightarrow\frac{a^2}{b}+\frac{b^2}{a}-a-b+\frac{b^2}{c}+\frac{c^2}{a}-\frac{b^2}{a}-c+a+b+c-\sqrt{3(ab+ac+bc)}\geq\)

\(\geq2((a-b)^2+(c-a)(c-b))\)

\(\Leftrightarrow(a-b)^2\left(\frac{1}{a}+\frac{1}{b}-2\right)+(c-a)(c-b)\left(\frac{1}{a}+\frac{b}{ac}-2\right)+a+b+c-\sqrt{3(ab+ac+bc)}\geq0\)

Đúng bởi \(\frac{1}{a}+\frac{1}{b}-2>0;\frac{1}{a}+\frac{b}{ac}-2\geq\frac{1}{a}+\frac{1}{a}-2>0\)

\(a+b+c-\sqrt{3(ab+ac+bc)}=\frac{(a-b)^2+(c-a)(c-b)}{a+b+c+\sqrt{3(ab+ac+bc)}}\geq0\)

BĐT đã được c/m. Vậy \(M_{Min}=\sqrt{3}-2\Leftrightarrow a=b=c=\dfrac{1}{\sqrt{3}}\)

P/s: Nhìn qua thấy ngon mà làm mới thấy thật sự là "choáng"

18 tháng 2 2020

Với x,y>0 ta cm: \(\frac{1}{x}+\frac{1}{y}\ge\frac{4}{x+y}\Leftrightarrow\frac{x+y}{xy}\ge\frac{4}{x+y}\Leftrightarrow\left(x+y\right)^2\ge4xy\Leftrightarrow\left(x-y\right)^2\ge0\) (luôn đúng)

=>\(\frac{1}{x+y}\le\frac{1}{4}\left(\frac{1}{x}+\frac{1}{y}\right)\)

ÁP dụng vào bài toán ta có: 

\(\frac{1}{a+b+2c}=\frac{1}{a+c+b+c}\le\frac{1}{4}\left(\frac{1}{a+c}+\frac{1}{b+c}\right)\)

\(\Rightarrow\frac{4ab}{a+b+2c}\le\frac{ab}{a+c}+\frac{ab}{b+c}\)

tương tự: \(\frac{4bc}{b+c+2a}\le\frac{bc}{a+b}+\frac{bc}{a+c};\frac{4ca}{c+a+2b}\le\frac{ca}{b+c}+\frac{ca}{a+b}\)

Cộng 3 bđt trên vế theo vế ta dc \(4\left(\frac{ab}{a+b+2c}+\frac{bc}{b+c+2a}+\frac{ca}{c+a+2b}\right)\le\frac{bc+ca}{a+b}+\frac{ab+ca}{b+c}+\frac{ab+bc}{a+c}=c+a+b\)

=>đpcm

Dấu "=" xảy ra <=> a=b=c

31 tháng 5 2021

\(\frac{ab}{a+b+2c}=\frac{ab}{\left(a+c\right)+\left(b+c\right)}\le\frac{1}{4}\left(\frac{ab}{a+c}+\frac{ab}{b+c}\right)\) 

Làm tương tự với 2 phân thức còn lại rồi cộng vào ra đpcm 

23 tháng 3 2017

mình chịu bó tay

24 tháng 3 2017

sao lại thế -.-